Derivative of (-1)^n/l'hopital's rule

  • Thread starter vinnie
  • Start date
  • Tags
    Derivative
In summary, the conversation discusses using the alternating series test to show that a given series converges. The first part of the test involves showing that the limit of the absolute value of the nth term of the series is 0. The second part involves determining if the sequence of the nth term is decreasing. It is noted that the sum of this particular series cannot be expressed in elementary terms. The use of L'Hopital's rule is found to be not applicable in this case.
  • #1
vinnie
23
0

Homework Statement



I am tryint to show that a series converges using the alternating series test. First, we show that 0<an+1<an and then that the limit of the expression equals 0. (the question says show that the series converges using the alternating series test, so I know it converges.)

Homework Equations



l'hopital's rule,

The Attempt at a Solution



the series is ((-1)^n)/(2sqrt(3n))

I need to use l'hopital's to show that the limit equals zero. The problem is, what is the derivative of (-1)^n if ln (-1) doesn't exist?
 
Physics news on Phys.org
  • #2
Hi Vinnie,

If a_n is the absolute value of the nth term (i.e., the factor being multiplied by (-1)^n), in this case 1/(2*sqrt(3n)), then part of the alternating series test asks that [tex]\lim_{n\to \infty} a_n = 0[/tex], so (-1)^n does not occur in the limit.
 
  • #3
so we would take the limit of 1/2srqrt(3n)?

how would we find the sum of this infinite series?
 
  • #4
vinnie said:
so we would take the limit of 1/2srqrt(3n)?
For the second part of alternating series test, yes. The other part of the test, as you have said, requires you to show that the sequence 1/(2*sqrt(3n)) (a_n as I defined previously) is decreasing. Review your notes.

how would we find the sum of this infinite series?
The alternating series test tells you the series converges, which is all the question required. This particular series' sum cannot be written in elementary terms. In mathematics, determining precisely what a quantity is may be a completely different task to simply showing that the quantity exists.
 
  • #5
Step 1: Determine a_n
Step 2: Determine if the SEQUENCE a_n is monotone decreasing

ie: The limit is 0 and it is infact decreasing a_n < a_(n-1)

Wikipedia should have more info on the alternating series test if this is not clear enough.

Note: Not sure why you want to use l'hospital, its not applicable.
 
  • #6
moo5003 said:
Step 1: Determine a_n
Step 2: Determine if the SEQUENCE a_n is monotone decreasing

ie: The limit is 0 and it is infact decreasing a_n < a_(n-1)

Wikipedia should have more info on the alternating series test if this is not clear enough.

Note: Not sure why you want to use l'hospital, its not applicable.

Now that I know about (-1)^n, I know it's not applicable.
 
  • #7
Unco said:
For the second part of alternating series test, yes. The other part of the test, as you have said, requires you to show that the sequence 1/(2*sqrt(3n)) (a_n as I defined previously) is decreasing. Review your notes.


The alternating series test tells you the series converges, which is all the question required. This particular series' sum cannot be written in elementary terms. In mathematics, determining precisely what a quantity is may be a completely different task to simply showing that the quantity exists.

I was speaking of the second part of the test. I know all about the first part. Thank you.
 
  • #8
To show 1/(2sqrt(3n)) is decreasing, you could take the derivative of 1/(2sqrt(3x)) and show that it is negative for positive x, or just note that since n is increasing 1/sqrt(n) must be decreasing.
 

1. What is the derivative of (-1)^n using l'Hopital's rule?

The derivative of (-1)^n using l'Hopital's rule is equal to 0. This is because the function (-1)^n is not continuous, and therefore, cannot be differentiated using traditional methods. However, l'Hopital's rule allows us to bypass this limitation and find the derivative of (-1)^n.

2. How does l'Hopital's rule work for finding the derivative of (-1)^n?

L'Hopital's rule states that if the limit of a fraction is in an indeterminate form (such as 0/0 or ∞/∞), then the limit of the fraction's derivative is equal to the derivative of the individual functions. In the case of (-1)^n, the limit of its derivative is equal to the derivative of the individual functions (-1) and n, which is equal to 0.

3. Can l'Hopital's rule be used for finding the derivative of any function?

No, l'Hopital's rule can only be used for finding the derivative of functions that are in an indeterminate form. It is not a general method for finding derivatives of all functions.

4. How do you apply l'Hopital's rule to find the derivative of (-1)^n?

To apply l'Hopital's rule, you need to rewrite the function in the form of a fraction and then take the derivative of the numerator and denominator separately. Then, take the limit of the derivative of the fraction as x approaches the desired value. In the case of (-1)^n, the desired value is often 0 or ∞.

5. Is l'Hopital's rule the only method for finding the derivative of (-1)^n?

No, there are other methods for finding the derivative of (-1)^n, such as using the definition of the derivative or using logarithmic differentiation. However, l'Hopital's rule is often the most efficient and straightforward method for finding the derivative of (-1)^n.

Similar threads

  • Calculus and Beyond Homework Help
Replies
1
Views
228
  • Calculus and Beyond Homework Help
Replies
2
Views
168
  • Calculus and Beyond Homework Help
Replies
17
Views
1K
  • Calculus and Beyond Homework Help
Replies
5
Views
976
  • Calculus and Beyond Homework Help
Replies
17
Views
591
Replies
1
Views
621
  • Calculus and Beyond Homework Help
Replies
2
Views
700
  • Calculus and Beyond Homework Help
Replies
14
Views
1K
  • Calculus and Beyond Homework Help
Replies
17
Views
1K
  • Calculus and Beyond Homework Help
Replies
23
Views
1K
Back
Top